A very classic number theory problem

Sdílet
Vložit
  • čas přidán 4. 08. 2024
  • Suggest a problem: forms.gle/ea7Pw7HcKePGB4my5
    Please Subscribe: czcams.com/users/michaelpennma...
    Merch: teespring.com/stores/michael-...
    Personal Website: www.michael-penn.net
    Randolph College Math: www.randolphcollege.edu/mathem...
    Randolph College Math and Science on Facebook: / randolph.science
    Research Gate profile: www.researchgate.net/profile/...
    Google Scholar profile: scholar.google.com/citations?...
    If you are going to use an ad-blocker, considering using brave and tipping me BAT!
    brave.com/sdp793
    Buy textbooks here and help me out: amzn.to/31Bj9ye
    Buy an amazon gift card and help me out: amzn.to/2PComAf
    Books I like:
    Sacred Mathematics: Japanese Temple Geometry: amzn.to/2ZIadH9
    Electricity and Magnetism for Mathematicians: amzn.to/2H8ePzL
    Abstract Algebra:
    Judson(online): abstract.ups.edu/
    Judson(print): amzn.to/2Xg92wD
    Dummit and Foote: amzn.to/2zYOrok
    Gallian: amzn.to/2zg4YEo
    Artin: amzn.to/2LQ8l7C
    Differential Forms:
    Bachman: amzn.to/2z9wljH
    Number Theory:
    Crisman(online): math.gordon.edu/ntic/
    Strayer: amzn.to/3bXwLah
    Andrews: amzn.to/2zWlOZ0
    Analysis:
    Abbot: amzn.to/3cwYtuF
    How to think about Analysis: amzn.to/2AIhwVm
    Calculus:
    OpenStax(online): openstax.org/subjects/math
    OpenStax Vol 1: amzn.to/2zlreN8
    OpenStax Vol 2: amzn.to/2TtwoxH
    OpenStax Vol 3: amzn.to/3bPJ3Bn
    My Filming Equipment:
    Camera: amzn.to/3kx2JzE
    Lense: amzn.to/2PFxPXA
    Audio Recorder: amzn.to/2XLzkaZ
    Microphones: amzn.to/3fJED0T
    Lights: amzn.to/2XHxRT0
    White Chalk: amzn.to/3ipu3Oh
    Color Chalk: amzn.to/2XL6eIJ

Komentáře • 150

  • @AlexBesogonov
    @AlexBesogonov Před 3 lety +40

    Thank you for a number theory problem that doesn't involve working in mod N!

  • @goodplacetostop2973
    @goodplacetostop2973 Před 3 lety +39

    12:50

  • @mcwulf25
    @mcwulf25 Před 3 lety +40

    Excellent stuff.
    The cherry on the cake was a solution p=19. So often there is no solution, or the solution is 0 or 1.

    • @bregottmannen2706
      @bregottmannen2706 Před 3 lety +2

      those aint even prime bro

    • @mcwulf25
      @mcwulf25 Před 3 lety +2

      @@bregottmannen2706 I know but in other problems Michael does the answers are often small numbers

  • @synaestheziac
    @synaestheziac Před 3 lety +38

    Great problem. I especially like how it can be solved using only high school math but would be very challenging for all but the strongest high school math students.

    • @cryfan6828
      @cryfan6828 Před 3 lety +4

      Absolutely. It’s really simply amazing

    • @praphael
      @praphael Před 2 lety

      It could also be solved by anyone who knows how to program, although you do not prove there are no solutions beyond a certain p, in practice on a question like this there won't be any for a "sufficiently large" p, probably < 100.
      von Neumann might have taken that approach, but using his head and not a calculator ...

    • @tollspiller2043
      @tollspiller2043 Před 3 měsíci

      @@praphael well, no, it could not be "solved" because "solving" requires proving that those are the only solutions, simply stating "probably p < 100" would get 0 points on any exam

    • @praphael
      @praphael Před 3 měsíci

      ​@@tollspiller2043 I wasn't suggesting thats what you would put on the exam. But if you were given this problem on an exam, where you were only allowed pen and paper, its extremely unlikely that there would be solutions beyond about p > ~100, probably even much smaller. The reason is because prime numbers cannot be generalized by a simple formula, which is part of what makes them so interesting. So if the answer were something like "infinitely many" with some restrictions to a certain class of primes, I would be hard pressed to come up with some closed form formula for actually generating them.
      And for very large prime, again it would be difficult for even good mathematicians to find them without the aid of a computer
      Lastly if you want to get technical, the question only asks you find them, not for a proof uniqueness. So I would definitely take issue with the "zero points".

  • @samuelmarger9031
    @samuelmarger9031 Před 3 lety +24

    Very cool question indeed.

  • @perappelgren948
    @perappelgren948 Před 2 lety +1

    Rewatching this again. Such great a video, Prof P! 👍👍

  • @TheEyalYemini
    @TheEyalYemini Před 3 lety +78

    Where do you learn this stuff? I am able to follow the explanation but I feel I would never be able to do this by myself…

  • @isaacwalters747
    @isaacwalters747 Před 3 lety +6

    A problem I would love to see you tackle, or maybe even a special case:
    The product from n=1 to N of (3+1/(a_n)), where each a_n is greater than 1 and of the form +1 or -1 modulo 6, is never an integer power of 2.
    The N=2 case is easy to show, as the product is bounded below and above by (3^2, 4^2), or (9,16), and there are no powers of two on that open interval.

  • @user-nr3yb3ki9p
    @user-nr3yb3ki9p Před 3 lety +1

    Thanks for your hard work and good videos 😊

  • @markohorstmann9637
    @markohorstmann9637 Před 3 lety +1

    Thanks for the fun videos! Love from Serbia❤️

  • @goblinkoma
    @goblinkoma Před 3 lety +46

    Did he mean "integer solutions" in the second hint? If not, someone pls explain i'm lost..

    • @Grizzly01
      @Grizzly01 Před 3 lety +22

      He did.
      An integral solution is one that all the unknown variables take only integer values.

    • @goblinkoma
      @goblinkoma Před 3 lety +1

      @@Grizzly01 ah ok thank you!

    • @schweinmachtbree1013
      @schweinmachtbree1013 Před 3 lety +19

      yup, "integral" is also an adjective meaning "which is an integer". so you could say that integral integrals are definite integrals from calculus whose values are integers :D

    • @dominickmancine6033
      @dominickmancine6033 Před 3 lety +1

      @@schweinmachtbree1013 But understanding this is not integral to solving the problem in the video. :)

    • @sayaksengupta4370
      @sayaksengupta4370 Před 3 lety

      @@schweinmachtbree1013 That is not always true. Integral does not always means an integer. But in this case it does.

  • @ChefSalad
    @ChefSalad Před 3 lety +12

    That second fact is missing a caveat. It's only true for polynomials with integer coefficients (I think only integers, anyway). For example, take x²+(2+2√3)x+1+2√3=0. One of it's roots is x=-1, an integer, but Δ=12, which is not a perfect square.

    • @reeeeeplease1178
      @reeeeeplease1178 Před 2 lety

      It should work if a,b,c are integers as you said since applying the quadratic formular and splitting the fraction leaves you with
      int/int + sqrt/int
      If you want this to be an integer (or even just a rational), the sqrt has to equal an integer
      As such, the discriminate has to be a perfect square

    • @elkincampos3804
      @elkincampos3804 Před 2 lety

      12 is a square in Z(square(3)). 12=(2*square(3))^2

  • @jordanweir7187
    @jordanweir7187 Před 3 lety +1

    Amazing problem, nice vid

  • @mrl9418
    @mrl9418 Před 3 lety +3

    This was a good one

  • @alonsoal6420
    @alonsoal6420 Před 3 lety +1

    Beautiful!

  • @MyOneFiftiethOfADollar
    @MyOneFiftiethOfADollar Před 3 lety +3

    If you examine the original discriminate of 4n^3 - 3, it is a perfect square for n=7 which also leads to p=19. So guessing and testing works which happens to be not so time consuming in this case.

    • @praphael
      @praphael Před 2 lety

      I'm not sure guess and check would be accepted though, because you haven't proved the uniquess of the solution.

  • @alaechoulli6111
    @alaechoulli6111 Před 3 lety

    You're the best!!

  • @joaomatheus294
    @joaomatheus294 Před 2 lety

    Beautiful question

  • @garydetlefs6095
    @garydetlefs6095 Před 2 lety

    I love your videos

  • @ianloree2784
    @ianloree2784 Před 3 lety +2

    Only one solution, very satisfying!

  • @curiosityxxx4305
    @curiosityxxx4305 Před 3 lety

    Plz make some log, e & trigonometry problems too.

  • @somasahu1234
    @somasahu1234 Před 2 lety +2

    Amazing

  • @peterquartararo3249
    @peterquartararo3249 Před 3 lety

    brilliant.!!!

  • @matteoanoffo1447
    @matteoanoffo1447 Před 3 lety +5

    You are Great. If you have to repost some old videos in a Better way i think you should make some algebra wich Is hard to find on CZcams. Thanks and Sorry for bad english

  • @tubamazouz
    @tubamazouz Před 3 lety

    Greatest !

  • @ijuhat19
    @ijuhat19 Před 2 lety +2

    The last part is a lot simpler if you just use p = mn - m + 1; setting m = 3 and n = 1 or 7 gives you p = 1 or 19 respectively.

    • @seroujghazarian6343
      @seroujghazarian6343 Před 2 lety

      1 is not prime

    • @ijuhat19
      @ijuhat19 Před 2 lety +1

      @@seroujghazarian6343 Well, yes, and you just exclude the p=1 solution for that reason.

  • @bktreesdoesmc8957
    @bktreesdoesmc8957 Před 3 lety +1

    May have found a different(but a little convoluted) way to solve the problem but with a similar idea.
    From p|n^2+n+1, we also have (n-1)|(p-1). Hence, p-1=k(n-1) and p>k(n-1). But we also have n^2+n+1>=p.
    Hence, n^2+n+1>=p>k(n-1). Thus, n^2+n+1>k(n-1), and hence (n^2-(k-1)n+k+1)>0, where k and n are natural numbers.
    Consider the equation n^2-(k-1)n+(k+1)=0
    For this equation to have no real roots (i.e, n^2-(k-1)n+(k+1)>0 is true for all real numbers(and hence natural numbers n), we must have D

  • @sarthak6605
    @sarthak6605 Před 3 lety +2

    Even good thing is if we replace p with any natural number, still the question is doable

  • @satyapalsingh4429
    @satyapalsingh4429 Před 3 lety +3

    My heart is filled with joy . Very good method of finding prime number p . God bless you .

  • @manucitomx
    @manucitomx Před 3 lety +8

    Wow!
    If ever it merited a backflip it was here.

    • @Tiqerboy
      @Tiqerboy Před 3 lety +2

      Would you rather be good at backflips or math?
      Michael Penn : Yes

  • @goodplacetostop2973
    @goodplacetostop2973 Před 3 lety +23

    HOMEWORK : A tightrope walker stands in the center of a rope of length 32 meters. Every minute she walks forward one meter with probability 3/4 and backward one meter with probability 1/4. What is the probability that she reaches the end in front of her before the end behind her?
    SOURCE : Guts Round of HMMT 2003

    • @pardeepgarg2640
      @pardeepgarg2640 Před 3 lety

      Bruh I don't understand what the question is this 🤣🤣🤣

    • @Thaplayer1209
      @Thaplayer1209 Před 3 lety +2

      My guts tell me it’s 3/4

    • @sirgog
      @sirgog Před 3 lety +12

      I like this problem.
      HINT:
      GP2S's choice of a power of 2 was deliberate.
      SOLUTION (not rigorous, this is an outline):
      Consider which occurs first: reaching 2 steps forward, or 2 steps back. After 2 steps, she is 9/16 to be 2 forward, 1/16 to be 2 back, and 6/16 to be back at the start. However if she is at position 0 (the centre), we can simply repeat this process. Note that Pr(she hovers near the centre for an indeterminate amount of time) approaches 0.
      This makes her 9/10 to reach +2 first, 1/10 to reach -2 first. Call a sequence of moves which continues until the walker ends up two steps forward or backward a second order move.
      Repeat this argument to see which occurs first: +4 or -4. See she is 81/82 to reach +4 first, 1/82 to reach -4 first (81/100 to reach +4 after 2 second order moves, 1/100 to end up -4 after 2 second order moves, 18/100 to return to the origin in which case we do 2 more second order moves). This is a third order move.
      Repeat third order moves to see which comes first: +8 or -8. 6561/6562 to be +8, 1/6562 to be -8
      Repeat and see she's 3^16/(3^16 + 1) to reach +16 before -16.
      TL:DR - don't play rigged casino games

    • @sjoerdo6988
      @sjoerdo6988 Před 3 lety +2

      My solution: We represent the possible positions by values of n from 0 to 32, such that n=0 represents the location of the end behind her, and n=32 represents the end in front of her. Currently, she is at n=16. Let pk be the probability of reaching n=32 before reaching n=0, starting from n=k.
      Clearly p0=0, and p32=1. For any value k from 1 to 31, it holds that:
      pk=1/4*p(k-1)+3/4*p(k+1). This, together with the boundary conditions results in p_k=(1-(1/3)^n)/(1-(1/3)^32).
      The probability of reaching the end before the start from n=16 is then (1-(1/3)^16)/(1-(1/3)^32), which is approximately 0.99999998. Is this answer correct?

    • @romajimamulo
      @romajimamulo Před 3 lety

      Nearly 1. 9^8/(9^8+1)
      I computed this by starting with the fact after two moves, you're either at +2, 0, or -2 from where you started. You can then move all the "probability of reaching the top" terms from your current position to see the probability in terms of going 2 up or 2 down. Then, you repeat the process, doubling the recursion formula until you get the probability of going 16 up or 16 down (and not stopping till you reach one of those)

  • @luisfelipe7351
    @luisfelipe7351 Před 2 lety

    a=Log[10,n/(n+1)]-Log[10,x/(x+1)] = n= prime x= position , a=0.12*Pi/x very close so having a we can have for a given prime the position n by NSolve

  • @pianochannel100
    @pianochannel100 Před 2 lety

    Number theory is some of my favorite math

  • @paulgillespie542
    @paulgillespie542 Před 2 lety

    I used the same factoring, but decided to show that n^2+n+1=0 has no integer solutions.

  • @blackmagicprod7039
    @blackmagicprod7039 Před 3 lety

    This video was nuts

  • @77Chester77
    @77Chester77 Před 2 lety

    Nice video!
    2:24 is it also possible, that (p-1) divides one of A or B?

    • @akanegally
      @akanegally Před 2 lety +1

      no it's not useful because p-1 is not necessarly a prime. Only the prime factors of p-1 divides one of A or B.

  • @alainbarnier1995
    @alainbarnier1995 Před 3 lety +1

    Trop beau :-)

  • @__zenon
    @__zenon Před 3 lety +3

    Why are the videos getting more and more silent? I can't turn the volume up anymore.

  • @beautifulworld6163
    @beautifulworld6163 Před 2 měsíci

    Thats exactly how i did. Fact that p is prime is helps a lot for the solution l.

  • @Oskar-zt9dc
    @Oskar-zt9dc Před 3 lety

    whats going on with the audio in the last vids?

  • @romajimamulo
    @romajimamulo Před 3 lety +10

    If you're going to stick with wider than 16:9, upload it without a black bar at the bottom so people with wider screens can appreciate it

  • @vishalramadoss668
    @vishalramadoss668 Před 2 lety

    Nice

  • @Smilesallnightlong
    @Smilesallnightlong Před 3 lety

    Is there an explanation/proof for the second hint anywhere? I feel a bit stupid for not understanding it.

    • @Nnm26
      @Nnm26 Před 3 lety

      integral solution = interger solution

  • @HGCFGHFHJ
    @HGCFGHFHJ Před 2 lety

    I MAY HAVE BETTER ONE AS IF U CAN SEE AT 2:16 after splitting gcd of n-1 and n2+n+1 term has gcd 1 or 3 by edl n2+n+1=n-1*n-2 +3 >gcd of (n-1,n2+n+1)=(n-1,3) staement of edalgorithm
    if u take their gcd as 1 u can simply equate p=n2+n+1 and p-1 to n-1 as p and p-1 also has gcd 1
    it gives no soln
    now if take gcd 3
    then put n=3m+1
    then u have got 9 as common then remaining has gcd 1so take p=3m2+3m+1and p-1 as 9m get m=2 >n=7>p=19
    i may have missed out some steps but if u r old learner u can think of it
    for this soln u need betyter understanding of gcd and edl and eda

  • @helo3827
    @helo3827 Před 3 lety

    4:16

  • @MrAmangandhi
    @MrAmangandhi Před 3 lety +1

    can anybody explain the proof to the homework part at 9:14

  • @davidbrisbane7206
    @davidbrisbane7206 Před 3 lety +2

    @Stephen Beck
    I put a bit more work into the solution and solved it with a different approach.
    Solve p² - p + 1 = n³, where p and n are integers. I.e. It is not assumed here that p is prime.
    So, p² - p + (1 - n³) = 0
    So, if we are to have any hope that p is an integer, then we require the discriminate D say to be an integer.
    Now D² = 1 - 4(1 - n³)
    So, 4n³ = D² + 3
    Now let n = x/4 and D = y/4, then
    4n³ = D² + 3 becomes x³/16 = y²/16 + 3
    So, x³ = y² + 48.
    This is a Mordell equation with known solution in integers of (x, y) = (4, 4), (4, -4), (28, 148) and (28, -148).
    This equates to the solutions (n, D) = (1, 1), (1, -1), (7, 37) and (7, -37) for the equation
    4n³ = D² + 3
    Now p² - p + 1 = n³ with D² = 4n³ - 3
    So, p = (1 ±D)/2
    Now, if ±D = 1, then p = 1
    If ±D = -1, then p = 0
    If ±D = 37, then p = 19
    If ±D = -37, then p = -18.
    These are the *only* integer values for p satisfying the equation p² - p + 1 = n³.
    So, the only prime value for p is p = 19.
    A point or two to note about this approach is that it doesn't assume p is prime. So, we lose the power of p being prime in the analysis, but we find all integer solutions for p, which happen to be the same ones Michael found, but his approach does not guarantee that he will find them. In his analysis, the non-prime integer values he found are really extraneous solutions. Of course, the above approach relies on a Mordell equation result, which I have not proven here.

  • @farklegriffen2624
    @farklegriffen2624 Před 3 lety +1

    New question, what primes make that prime?

  • @goodplacetostart9099
    @goodplacetostart9099 Před 3 lety

    Good Place To Start At 1:27

  • @Douae1111
    @Douae1111 Před 3 měsíci

    what does he mean by integral solutions ?

  • @EternalLoveAnkh
    @EternalLoveAnkh Před 2 lety +1

    I have a better question. Which cubes make that prime?
    RJ

  • @crep50
    @crep50 Před 3 lety +3

    It might look like an odd question, but it can actually be represented by p^2 - p^1 + p^0

  • @rosiefay7283
    @rosiefay7283 Před 3 lety

    Wgy do you describe many of those numbers as perfect?

    • @sjs260563
      @sjs260563 Před 3 lety

      Perfect number, a positive integer that is equal to the sum of its proper divisors

  • @frankrentt2225
    @frankrentt2225 Před 3 lety

    Someone know why only check n as a variable, and no the other case (n as a constant)?

  • @KarlDeux
    @KarlDeux Před 2 lety

    Also, p cannot divide n-1 because n-1 is even.
    p is not 2, so p is odd, so p²-p+1=n is odd as well.

  • @dblaze23
    @dblaze23 Před 3 lety +1

    10:43
    We can put an X next to it.
    Proceeds to put an *
    Awesome video though

  • @stevenmellemans7215
    @stevenmellemans7215 Před 3 lety

    Someone needs to increase the circumference of the lotion distribution 😄

  • @mcwulf25
    @mcwulf25 Před 2 lety

    ...unless p | GCD(a,b) in which case p | a AND p | b.

  • @rogeriojunior9459
    @rogeriojunior9459 Před rokem

    Very good 19

  • @peeyushagarwal5889
    @peeyushagarwal5889 Před 3 lety

    at 2:58, how is pn^3, p cannot be smaller than n.

    • @oliverherskovits7927
      @oliverherskovits7927 Před 3 lety +1

      This inequality is due to the divisibility argument. If a number divides another number, the divisor must be less than or equal to the multiple (if both are positive). What u noticed is essentially the contradiction he builds next

    • @bonjour7209
      @bonjour7209 Před 2 lety

      That was his point

  • @jarikosonen4079
    @jarikosonen4079 Před 3 lety +1

    It looks kind of luck that p=19 prime number, but it looks proven there is no other ones... I hope I can learn something about this, even possibly not ever needed.

  • @particleonazock2246
    @particleonazock2246 Před 3 lety

    AMOGUS!

  • @Calculus58
    @Calculus58 Před 11 měsíci

    You should check your answer

  • @HoSza1
    @HoSza1 Před 2 lety

    C, D, E, F, G, A, B
    Period.

  • @Max-sg5tz
    @Max-sg5tz Před 3 lety +1

    Hello guys,
    I am asking you for advice:
    With his videos, Michael got me really interested in mathematics (or better: solving mathematical-olympiad type of questions). But as a newbie, I don't have all these different types of "tricks" in my repertoire, so I am often unable to solve them. I have started creating a list with all the different tricks/methods that Michael and the other Math-CZcamsrs use, and it has been really helpful, but it will take ages to complete it. Do you have any literature tips (also websites) where these "mathematical tricks" are illustrated in a systematic way? Any other pieces of advice to make quick advances in math are also welcomed.
    Thanks in advance.

    • @davidbrisbane7206
      @davidbrisbane7206 Před 3 lety +1

      You can buy books on Putnam problems & solutions. They tend to be a bit expensive, but they usually teach you the techniques you need to solve competition problems.

  • @utsav8981
    @utsav8981 Před 3 lety +2

    What did he do just after 7:48? I'm lost
    Also, Why did he choose to make that stuff less than D?

    • @thiagozanfolin9349
      @thiagozanfolin9349 Před 3 lety +1

      So most of the times that you get an expression and wants to see if it is a perfect square(ps),you can try fitting It between to consecutive ps because if that happens,you are done.So,he noticed that for every m,Dq^2

    • @henk7747
      @henk7747 Před 3 lety

      In order to show something is not a perfect square you can bound it between 2 squares. For example if you know 16 < x < 25, it's clear that x can not be a square.

    • @robertveith6383
      @robertveith6383 Před 3 lety

      @ henk -- You left out that you "bound it between two *consecutive perfect* squares."

    • @MrAmangandhi
      @MrAmangandhi Před 3 lety

      @@thiagozanfolin9349 can you explain the proof for why D

    • @thiagozanfolin9349
      @thiagozanfolin9349 Před 3 lety +1

      @@MrAmangandhi for sure!
      So we have D0
      The discriminant is 0.
      Just make the graph of 2m^2 -4m +7 and you will notice it imediatally!

  • @Reliquancy
    @Reliquancy Před 3 lety

    There’s some kind of unpleasant high frequency buzzing in the video.

  • @kenbrohere
    @kenbrohere Před 3 lety +1

    I was close. My guess was 23.

  • @rodrigojose3369
    @rodrigojose3369 Před 3 lety

    find all integers "x" such that x² + 4 is a cube

  • @kenbrohere
    @kenbrohere Před 3 lety

    Yeah, you're gonna need this.

  • @tommasomarchioni2133
    @tommasomarchioni2133 Před 3 lety +1

    Aren't 0 and 1 primes?!? Someone explain

    • @TayTaaay
      @TayTaaay Před 3 lety

      A prime number is divisible by itself and 1 so a prime number has 2 divisors.
      When we look at 0 then we can find that 0 is divisible by all real numbers (besides itself of course) because there is atleast one q such that 0.q = 0 therefore we know that 0 has infinite divisors.
      For 1 we know that it only has one divisor, namely just 1 itself. In order for 1 to be a prime number it has to have 2 divisors but it doesn’t so 1 is not prime.

  • @olau5478
    @olau5478 Před 3 lety

    0:52 what is integral solutions? is it supposed to be integer?

  • @andraspongracz5996
    @andraspongracz5996 Před 3 lety

    Nice one. In fact, it would have been more precise to emphasize that p is a POSITIVE prime. That is a big extra condition that you make heavy use of.
    I think the problem can be fully solved without that extra condition as well. My first idea when seeing the expression p^2-p+1 was to use Eisenstein numbers (Eulerian numbers), as p^2-p+1=(p+w)(p+w^2) with w being a primitive third root of unity. I think it works, there are a number of cases to check (like 18 major ones), but they all lead to relatively simple systems of equations to solve. The main point is of course that p+w and p+w^2 are either coprime or their gcd is sqrt{-3}=1+2w, which is an Eisenstein prime. So either p+w is a cube of an Eisenstein integer multiplied by an Eisenstein unit, or the same goes for (p+w)(1+2w) and (p+w^2)/(1+2w) or the same goes for (p+w^2)(1+2w) and (p+w)/(1+2w).

  • @davidbrisbane7206
    @davidbrisbane7206 Před 3 lety +4

    We could hope that p is a small prime and try p = 2,3,5,7,11,13,17,19 etc and stop when we find a value of p that satisfies the equation p² - p + 1 = n³ where n ∈ ℤ⁺.
    As p² - p + 1 = n³ is a quadratic equation in p, then we know that p can have at most *two* distinct real values.
    When we get to p = 19 by the trial and error approach, then p² - p + 1 = 343 = 7³.
    So p = 19 is a solution, and it is pime.
    As we have found one real value satisfying the quadratic, we know that the other value (root) satifying the quadratic is real too.
    Now we need to find the other value to the quadratic and check if it is prime or not.
    Now let (p - 19)(p - a), where a ∈ ℝ and
    So, (p - 19)(p - a) = p² - p - 342
    where a is the other root of the quadratic.
    Now, (p - 19)(p - a)
    = p² - (19 + a)p + 19a
    = p² - p - 342
    So, 19a = -342 and 1 = 19 + a
    So, in either case, a = -18 is the other real solution and it is *not* prime.
    Hence, the only value of p being prime resulting in p² - p + 1 = n³ is p = 19.

    • @ogasdiaz
      @ogasdiaz Před 3 lety

      p can have at most two distinct values for each cube. Not overall.

    • @jeffreykornhauser9063
      @jeffreykornhauser9063 Před 3 lety +3

      This is not correct. Quadratics do have at most 2 real solutions, but n can be any positive integer here. So really, all you essentially did was prove that n=19 and n= -18 are the only real solutions to the quadratic equation p^2-p+1=7^3. But what about p^2+p-1=8^3, p^2+p-1=9^3, so on and so forth.

    • @davidbrisbane7206
      @davidbrisbane7206 Před 3 lety

      @@jeffreykornhauser9063
      What I have proven is that there are no more possible solutions.

    • @davidbrisbane7206
      @davidbrisbane7206 Před 3 lety

      @@ogasdiaz
      I'll need to check by supposing we found another cube value, say m³, such that p² - p + 1 = m^3 leads to a different set of solutions. Although, having said that, Michael proved in the end that there was only two possible value of p such that p² - p + 1 is a cube and only one of these was prime.

    • @stephenbeck7222
      @stephenbeck7222 Před 3 lety

      David Brisbane Michael found 4 values of p possible: -18, 0, 1, and 19, only one of which was prime. But there are surely more values of p to solve the original expression, since he used the primeness of p very early in the proof to restrict the relationship between p and n.

  • @jiyoungpark6233
    @jiyoungpark6233 Před 3 lety

    p = 1

  • @johnthevampire819
    @johnthevampire819 Před 2 lety

    If p=4 a=2 and b=2 then we have that 4|2*2 but 4|2 is false. Correct me if I am wrong but it seems that one of the facts doesn't work all the time.

  • @holyshit922
    @holyshit922 Před měsícem

    p = 19

  • @tonmoybhowmik8670
    @tonmoybhowmik8670 Před 3 lety

    I want to send you a problem

    • @aahaanchawla5393
      @aahaanchawla5393 Před 3 lety

      use the google form in the description

    • @helo3827
      @helo3827 Před 3 lety

      email him, I used the google form and never worked, but if I emailed, he did all of the videos(including this one)

    • @tonmoybhowmik8670
      @tonmoybhowmik8670 Před 3 lety

      @@helo3827 Can I get his Email address....? 😀

  • @arimermelstein9167
    @arimermelstein9167 Před 3 lety +5

    Stated more formally:
    If a quadratic polynomial has integral solutions, then its discriminant is a perfect square.
    The converse is not true.

    • @skylardeslypere9909
      @skylardeslypere9909 Před 3 lety +3

      "If a quadratic polynomial _with integer coefficients_ has integral solutions, then its discriminant is a perfect square"
      would be more correct

    • @kingkartabyo6206
      @kingkartabyo6206 Před 3 lety +1

      This partial converse is true: if a monic quadratic with integer coefficients has a perfect square discriminant, it has both roots integer.
      It simply means ax^2+bx+c=0 with b,c integers, a=1, and of course D=b^2-4ac is square.

    • @skylardeslypere9909
      @skylardeslypere9909 Před 3 lety +1

      @@kingkartabyo6206 the (pseudo)proof is kinda fun to think about.
      We know that b and b² have the same parity. Also b²-4ac has the same parity as b² (and thus) b because 4ac is even.
      Since sqrt(b²-4ac) is an integer, it ALSO has the same parity as b.
      Now, either b and sqrtD are odd, making their sum/difference even, which can be divided by 2a = 2, resulting in x1 and x2 integers.
      If b is even, then -b±sqrtD is obviously even, making x1 and x2 integers. QED

    • @kingkartabyo6206
      @kingkartabyo6206 Před 3 lety +1

      @@skylardeslypere9909 This is perfect! Well done

  • @playgroundgames3667
    @playgroundgames3667 Před 3 lety

    p^ = 17 | 17 + 1 = 18 | 17^ = 289 | 289 - 18 = 271 | the square root of 271 = 90 which is a perfect cube. 🔥🔥

    • @robertveith6383
      @robertveith6383 Před 3 lety

      You have many errors in that line. I do not know why everything was typed on one line.
      There are missing characters, run-ons, and/or incomplete/false statements.

  • @nguyenthanh-tm6dz
    @nguyenthanh-tm6dz Před 3 lety

    The unadvised felony surgically subtract because bar bodily welcome a a sleepy link. tart, jumpy passbook